Is Every Differential 1-Form on a Line the Differential of Some Function?

Click For Summary
Every differential 1-form on a line can be expressed as the differential of some function, specifically in the form of ω = g(x) dx. To demonstrate this, one must find a function G(x) such that ω = dG. The discussion highlights the importance of integration in this process. Participants emphasize the need to adhere to forum rules regarding providing complete solutions. Understanding the relationship between differential forms and functions is crucial in solving this problem.
Abhishek11235
Messages
174
Reaction score
39

Homework Statement


This problem is from V.I Arnold's book Mathematics of Classical Mechanics.
Q) Show that every differential 1-form on line is differential of some function

Homework Equations


The differential of any function is
$$df_{x}(\psi): TM_{x} \rightarrow R$$

The Attempt at a Solution



The tangent to line is line itself. The differential 1-form is ##dy-dx=0##. Here I am struct. I don't know how to find out the differential. Can anyone help?
 

Attachments

  • Screenshot_2019-02-01-20-59-12.png
    Screenshot_2019-02-01-20-59-12.png
    37.5 KB · Views: 323
Physics news on Phys.org
Abhishek11235 said:
The tangent to line is line itself.
No.

Abhishek11235 said:
The differential 1-form is dy−dx=0dy−dx=0dy-dx=0.
No. A general differential 1-form is of the form ##\omega = g(x) dx##. You have to show that there exists a function ##G(x)## such that ##\omega = dG##.
 
Orodruin said:
No.No. A general differential 1-form is of the form ##\omega = g(x) dx##. You have to show that there exists a function ##G(x)## such that ##\omega = dG##.

That means I have to integrate. Can you provide me solution?
 
Abhishek11235 said:
Can you provide me solution?
That would violate the forum rules, which you would realize if you had bothered reading them.
 
Question: A clock's minute hand has length 4 and its hour hand has length 3. What is the distance between the tips at the moment when it is increasing most rapidly?(Putnam Exam Question) Answer: Making assumption that both the hands moves at constant angular velocities, the answer is ## \sqrt{7} .## But don't you think this assumption is somewhat doubtful and wrong?

Similar threads

  • · Replies 2 ·
Replies
2
Views
2K
  • · Replies 6 ·
Replies
6
Views
3K
  • · Replies 22 ·
Replies
22
Views
3K
  • · Replies 10 ·
Replies
10
Views
3K
  • · Replies 1 ·
Replies
1
Views
1K
  • · Replies 2 ·
Replies
2
Views
2K
  • · Replies 1 ·
Replies
1
Views
1K
  • · Replies 6 ·
Replies
6
Views
2K
  • · Replies 39 ·
2
Replies
39
Views
6K
  • · Replies 5 ·
Replies
5
Views
2K